Gradient and curvilinear coordinates

Click For Summary
SUMMARY

The discussion focuses on proving the relationship ##\nabla u_i \cdot \frac{\partial \vec r}{\partial u_i} = \delta_{ij}## in the context of curvilinear coordinates. The gradient in curvilinear coordinates is expressed as ##\nabla \phi = \sum_{i=1}^3 \vec e_i \frac{1}{h_i} \frac{\partial \phi}{\partial u_i}##. The participants explore the orthogonality of gradients and partial derivatives, particularly in non-orthogonal systems, and clarify that the partial derivative ##\partial/\partial u_j## treats other coordinates as constants. The discussion concludes with insights on the dependency of unit vectors in cylindrical coordinates.

PREREQUISITES
  • Understanding of curvilinear coordinates
  • Familiarity with gradient operations in vector calculus
  • Knowledge of partial derivatives and their properties
  • Basic concepts of cylindrical coordinate systems
NEXT STEPS
  • Study the derivation of gradients in curvilinear coordinates
  • Explore the properties of partial derivatives in multivariable calculus
  • Learn about the relationship between unit vectors in cylindrical coordinates
  • Investigate applications of curvilinear coordinates in physics and engineering
USEFUL FOR

Students and professionals in mathematics, physics, and engineering who are working with vector calculus and curvilinear coordinate systems.

Incand
Messages
334
Reaction score
47

Homework Statement


Show that ##\nabla u_i \cdot \frac{\partial \vec r}{\partial u_i} = \delta_{ij}##.

(##u_i## is assumed to be a generalized coordinate.)

Homework Equations


Gradient in curvilinear coordinates
##\nabla \phi = \sum_{i=1}^3 \vec e_i \frac{1}{h_i} \frac{\partial \phi}{\partial u_i}##

The Attempt at a Solution


So what I need to show is that the gradient is orthogonal to the (other two) partial derivatives (but it doesn't have to be parallel to the third since the system doesn't have to be orthogonal!).
The expression can then be written as
##\sum_{k=1}^3 \left( \vec e_k \frac{1}{h_k} \frac{\partial u_i}{\partial u_k} \right) \cdot \frac{\partial \vec r}{\partial u_j} = \sum_{k=1}^3 \frac{1}{h_k}\frac{\partial u_i}{\partial u_k}\frac{\partial r_k}{\partial u_j} ##.
I'm thinking I could use the chain rule here but I don't seem to get anywhere. Another thing of note is that ##\frac{1}{h_k}\frac{\partial \vec r}{\partial u_k} = \vec e_k## which may be of use somewhere.
 
Physics news on Phys.org
You can substitue ##\partial \vec{r}/\partial u_j = h_j \vec{e}_j## in your expression.
 
  • Like
Likes Incand
fzero said:
You can substitue ##\partial \vec{r}/\partial u_j = h_j \vec{e}_j## in your expression.
Thanks!
##\sum_{k=1}^3 \left( \vec e_k \frac{1}{h_k}\frac{\partial u_i}{\partial u_k} \right) \cdot \frac{h_j}{h_j} \frac{\partial \vec r}{\partial u_j} = \sum_{k=1}^3 \left( \vec e_k \frac{1}{h_k}\frac{\partial u_i}{\partial u_k} \right) \cdot h_j \vec e_j = \frac{\partial u_i}{\partial u_j}##.
Which is obviously ##1## for ##i=j## but how do I know that it's zero when ##i\ne j##? For me this isn't obvious. In a cylindrical coordinate system the unit vector ##\hat \rho## is dependent on ##\hat \phi## and the other way around so I don't see why the coordinates them self can't be dependent.
 
Incand said:
Thanks!
##\sum_{k=1}^3 \left( \vec e_k \frac{1}{h_k}\frac{\partial u_i}{\partial u_k} \right) \cdot \frac{h_j}{h_j} \frac{\partial \vec r}{\partial u_j} = \sum_{k=1}^3 \left( \vec e_k \frac{1}{h_k}\frac{\partial u_i}{\partial u_k} \right) \cdot h_j \vec e_j = \frac{\partial u_i}{\partial u_j}##.
Which is obviously ##1## for ##i=j## but how do I know that it's zero when ##i\ne j##? For me this isn't obvious. In a cylindrical coordinate system the unit vector ##\hat \rho## is dependent on ##\hat \phi## and the other way around so I don't see why the coordinates them self can't be dependent.

The partial derivative ##\partial/\partial u_j## is defined so that ##u_{i\neq j}## are treated as constants for the purpose of taking the derivative.
 
  • Like
Likes Incand
fzero said:
The partial derivative ##\partial/\partial u_j## is defined so that ##u_{i\neq j}## are treated as constants for the purpose of taking the derivative.
Right, I wasn't thinking clearly, Thanks!
 
Question: A clock's minute hand has length 4 and its hour hand has length 3. What is the distance between the tips at the moment when it is increasing most rapidly?(Putnam Exam Question) Answer: Making assumption that both the hands moves at constant angular velocities, the answer is ## \sqrt{7} .## But don't you think this assumption is somewhat doubtful and wrong?

Similar threads

Replies
2
Views
1K
  • · Replies 9 ·
Replies
9
Views
2K
  • · Replies 3 ·
Replies
3
Views
3K
  • · Replies 3 ·
Replies
3
Views
2K
  • · Replies 10 ·
Replies
10
Views
615
  • · Replies 18 ·
Replies
18
Views
2K
  • · Replies 1 ·
Replies
1
Views
2K
  • · Replies 14 ·
Replies
14
Views
2K
  • · Replies 5 ·
Replies
5
Views
3K
  • · Replies 8 ·
Replies
8
Views
2K